Last visit was: 24 Apr 2024, 16:38 It is currently 24 Apr 2024, 16:38

Close
GMAT Club Daily Prep
Thank you for using the timer - this advanced tool can estimate your performance and suggest more practice questions. We have subscribed you to Daily Prep Questions via email.

Customized
for You

we will pick new questions that match your level based on your Timer History

Track
Your Progress

every week, we’ll send you an estimated GMAT score based on your performance

Practice
Pays

we will pick new questions that match your level based on your Timer History
Not interested in getting valuable practice questions and articles delivered to your email? No problem, unsubscribe here.
Close
Request Expert Reply
Confirm Cancel
SORT BY:
Kudos
Tags:
Show Tags
Hide Tags
Manager
Manager
Joined: 28 Sep 2016
Status:EAT SLEEP GMAT REPEAT!
Posts: 114
Own Kudos [?]: 183 [48]
Given Kudos: 143
Location: India
Send PM
Most Helpful Reply
GMAT Instructor
Joined: 01 Jul 2017
Posts: 89
Own Kudos [?]: 1430 [30]
Given Kudos: 11
Location: United States
Concentration: Leadership, Organizational Behavior
Send PM
General Discussion
Math Expert
Joined: 02 Sep 2009
Posts: 92900
Own Kudos [?]: 618811 [1]
Given Kudos: 81588
Send PM
Tutor
Joined: 04 Aug 2010
Posts: 1315
Own Kudos [?]: 3136 [1]
Given Kudos: 9
Schools:Dartmouth College
Send PM
Re: Pundit: By passing the recent tax bill, the Republicans have likely [#permalink]
1
Kudos
Expert Reply
Mo2men wrote:
GMATGuruNY

Why is "C" wrong? how to negate it ?

Thanks


C, negated:
The tax bill is less popular with likely voters than with unlikely voters.
In suggesting that the tax bill is unpopular with likely voters, this negation STRENGTHENS the conclusion that Republicans who passed the bill have likely thrown away their congressional majority in the upcoming election.
Since the correct negation must invalidate the conclusion, eliminate C.
Tutor
Joined: 04 Aug 2010
Posts: 1315
Own Kudos [?]: 3136 [1]
Given Kudos: 9
Schools:Dartmouth College
Send PM
Re: Pundit: By passing the recent tax bill, the Republicans have likely [#permalink]
1
Kudos
Expert Reply
Mo2men wrote:
how Choice D can't be an answer without applying negation test? I did not get your explanation in choice D.


An assumption is a statement that MUST BE TRUE for the conclusion to hold.
D implies the following:
To conclude that Republicans are trailing in the polls because of the tax bill, IT MUST BE TRUE that a majority of likely voters oppose the bill.
Not so.
Republican candidates could be trailing in the polls simply because 25% of likely voters oppose the bill.
Since D is not a statement that must be true, eliminate D.
Tutor
Joined: 04 Aug 2010
Posts: 1315
Own Kudos [?]: 3136 [1]
Given Kudos: 9
Schools:Dartmouth College
Send PM
Re: Pundit: By passing the recent tax bill, the Republicans have likely [#permalink]
1
Kudos
Expert Reply
PREMISE:
In polling since the passage of a recent tax bill, Republicans have trailed their competitors by 8 to 10 percentage points.
CONCLUSION:
By passing the recent tax bill, Republicans have thrown away their majority in the upcoming election.

The argument uses the POLLING to support the conclusion.
It ASSUMES -- without any evidence -- that the Republicans' negative poll numbers are DUE TO the passage of the tax bill.
B, negated:
The poll numbers prior to the tax bill’s passing were essentially identical to those taken after the tax bill passed.
Here, Republicans were trailing by 8 to 10 percentage points BEFORE the passage of the tax bill, trashing the assumption in red and invalidating the conclusion that the tax bill will be responsible for the Republicans' loss in the upcoming election.

GMAT Instructor
Joined: 01 Jul 2017
Posts: 89
Own Kudos [?]: 1430 [1]
Given Kudos: 11
Location: United States
Concentration: Leadership, Organizational Behavior
Send PM
Pundit: By passing the recent tax bill, the Republicans have likely [#permalink]
1
Bookmarks
Expert Reply
Mo2men wrote:
Dear Aaron,

Thanks for your great explanation.

Can you please expand more about how Choice D can't be an answer without applying negation test? I did not get your explanation in choice D.

Thanks in advance

Thank you for your question! Here is a more complete analysis of answer choice "D". For the convenience of other people studying for the GMAT, I also updated my original response so that everything is all in one place.

"D" is a very common wrong answer. When you contrast "D" with "C", it sounds like "D" isn't far from the mark. This answer choice strengthens the argument by implying that, on average, likely voters do not support the tax bill. But this is not a regular "Strengthen" question. We need to focus on what the question is actually asking: we are looking for an assumption necessary for the conclusion to function. While "D" helps the argument, it is not necessary for the argument to be valid. Think about it: what if 10% of the voters don't like the tax bill, 9% like it, and 81% don't care at all? A higher percentage of voters might not like the tax bill, but we still don't know if the tax bill even matters to the majority of the public! Additionally, while likely voters might not like the tax bill, it is equally possible that the polling results are due to something else. What if the Republicans in this question were already behind in the polls? What if they were going to lose the majority anyway? If this were the case, then the pundit's assumption that the reaction to the tax bill caused the poll results (and will cause problems with the election down the road) falls apart. "D" is useful, but isn't required by the argument. Get rid of it.

A parallel argument might be instructive here. Imagine if the problem said, "Candidate A has really bad facial acne. Most people don't like facial acne. Therefore, Candidate A is going to lose the election." Yes, it might be true that most people dislike zits. But we still can't assume a causal link between acne and elections. Ugly candidates get elected to office all the time! In like manner, just because people don't like the bill, this still doesn't mean that this is what is causing the polling numbers to shift. The argument needs something more... something to show that the passing of the tax bill truly changed how people feel about the candidates. That is what the argument requires, and answer choice "D" doesn't quite get there.

Here is now the full explanation, complete with these updates:
https://gmatclub.com/forum/pundit-by-pa ... l#p2085899
Director
Director
Joined: 05 Mar 2015
Posts: 852
Own Kudos [?]: 860 [1]
Given Kudos: 45
Send PM
Re: Pundit: By passing the recent tax bill, the Republicans have likely [#permalink]
Adi93 wrote:
Pundit: By passing the recent tax bill, the Republicans have likely thrown away their congressional majority in the upcoming election. In polling ever since the bill was passed into law, Republican congressmen have consistently trailed their closest competitors by an average of 8 to 10 percentage points.

Which of the following is an assumption on which the pundit’s argument relies?

A. None of the Republican congressmen’s closest competitors were in favor of the tax bill.
B. The poll numbers prior to the tax bill’s passing were substantially different than those taken after the tax bill passed.
C. The tax bill is no less popular with likely voters than it is with unlikely voters.
D. A higher percentage of likely voters oppose the new tax bill than support it.
E. The tax bill has been the most unpopular piece of legislation passed by the Republicans since the last election.



A. None of the Republican congressmen’s closest competitors were in favor of the tax bill.
B. The poll numbers prior to the tax bill’s passing were substantially different than those taken after the tax bill passed.
C. The tax bill is no less popular with likely voters than it is with unlikely voters.
D. A higher percentage of likely voters oppose the new tax bill than support it.
E. The tax bill has been the most unpopular piece of legislation passed by the Republicans since the last election.[/quote]

Ans B
RSM Erasmus Moderator
Joined: 26 Mar 2013
Posts: 2461
Own Kudos [?]: 1360 [0]
Given Kudos: 641
Concentration: Operations, Strategy
Schools: Erasmus (II)
Send PM
Re: Pundit: By passing the recent tax bill, the Republicans have likely [#permalink]
Adi93 wrote:
Pundit: By passing the recent tax bill, the Republicans have likely thrown away their congressional majority in the upcoming election. In polling ever since the bill was passed into law, Republican congressmen have consistently trailed their closest competitors by an average of 8 to 10 percentage points.

Which of the following is an assumption on which the pundit’s argument relies?

A. None of the Republican congressmen’s closest competitors were in favor of the tax bill.
B. The poll numbers prior to the tax bill’s passing were substantially different than those taken after the tax bill passed.
C. The tax bill is no less popular with likely voters than it is with unlikely voters.
D. A higher percentage of likely voters oppose the new tax bill than support it.
E. The tax bill has been the most unpopular piece of legislation passed by the Republicans since the last election.


GMATGuruNY

Why is "C" wrong? how to negate it ?

Thanks
RSM Erasmus Moderator
Joined: 26 Mar 2013
Posts: 2461
Own Kudos [?]: 1360 [0]
Given Kudos: 641
Concentration: Operations, Strategy
Schools: Erasmus (II)
Send PM
Re: Pundit: By passing the recent tax bill, the Republicans have likely [#permalink]
AaronPond wrote:

"D" is a very common wrong answer. When you contrast "D" with "C", it sounds like "D" isn't far from the mark. This answer choice implies that, on average, likely voters do not support the tax bill. However, we need to focus on what the question is actually asking: we are looking for an assumption necessary for the conclusion to function. Even though likely voters don't like the tax bill, it is equally possible that the polling results are due to something else. The pundit inappropriately draws a causal link between the poll results and the tax bill. What if the Republicans in this question were already behind in the polls? The pundit's argument falls apart. "D" isn't required by the argument. Get rid of it.


Dear Aaron,

Thanks for your great explanation.

Can you please expand more about how Choice D can't be an answer without applying negation test? I did not get your explanation in choice D.

Thanks in advance
RSM Erasmus Moderator
Joined: 26 Mar 2013
Posts: 2461
Own Kudos [?]: 1360 [0]
Given Kudos: 641
Concentration: Operations, Strategy
Schools: Erasmus (II)
Send PM
Re: Pundit: By passing the recent tax bill, the Republicans have likely [#permalink]
GMATGuruNY wrote:
Mo2men wrote:
how Choice D can't be an answer without applying negation test? I did not get your explanation in choice D.


An assumption is a statement that MUST BE TRUE for the conclusion to hold.
D implies the following:
To conclude that Republicans are trailing in the polls because of the tax bill, IT MUST BE TRUE that a majority of likely voters oppose the bill.
Not so.
Republican candidates could be trailing in the polls simply because 25% of likely voters oppose the bill.
Since D is not a statement that must be true, eliminate D.


Dear GMATGuruNY

The conclusion here is that "Republicans have likely thrown away their congressional majority in the upcoming election" but it is that "Republicans are trailing in the polls because of the tax bill". Does not this change the way to look into D?
Tutor
Joined: 04 Aug 2010
Posts: 1315
Own Kudos [?]: 3136 [0]
Given Kudos: 9
Schools:Dartmouth College
Send PM
Re: Pundit: By passing the recent tax bill, the Republicans have likely [#permalink]
Expert Reply
Mo2men wrote:
GMATGuruNY wrote:
Mo2men wrote:
how Choice D can't be an answer without applying negation test? I did not get your explanation in choice D.


An assumption is a statement that MUST BE TRUE for the conclusion to hold.
D implies the following:
To conclude that Republicans are trailing in the polls because of the tax bill, IT MUST BE TRUE that a majority of likely voters oppose the bill.
Not so.
Republican candidates could be trailing in the polls simply because 25% of likely voters oppose the bill.
Since D is not a statement that must be true, eliminate D.


Dear GMATGuruNY

The conclusion here is that "Republicans have likely thrown away their congressional majority in the upcoming election" but it is that "Republicans are trailing in the polls because of the tax bill". Does not this change the way to look into D?


The conclusion is that Republicans have likely thrown away their majority by PASSING THE TAX BILL.
In other words, the argument links passage of the bill to the negative poll results and to the probability of losing in the next election.
Clearly, Republicans could be trailing in the polls and could lose their majority even if only 25% of voters oppose the bill.
Thus, D is not a statement that MUST BE TRUE for the conclusion to hold.
Senior Manager
Senior Manager
Joined: 09 Mar 2017
Posts: 369
Own Kudos [?]: 823 [0]
Given Kudos: 646
Location: India
Concentration: Marketing, Organizational Behavior
WE:Information Technology (Computer Software)
Send PM
Pundit: By passing the recent tax bill, the Republicans have likely [#permalink]
Bunuel wrote:
Adi93 wrote:
Pundit: By passing the recent tax bill, the Republicans have likely thrown away their congressional majority in the upcoming election. In polling ever since the bill was passed into law, Republican congressmen have consistently trailed their closest competitors by an average of 8 to 10 percentage points.

Which of the following is an assumption on which the pundit’s argument relies?

A. None of the Republican congressmen’s closest competitors were in favor of the tax bill.
B. The poll numbers prior to the tax bill’s passing were substantially different than those taken after the tax bill passed.
C. The tax bill is no less popular with likely voters than it is with unlikely voters.
D. A higher percentage of likely voters oppose the new tax bill than support it.
E. The tax bill has been the most unpopular piece of legislation passed by the Republicans since the last election.


VERITAS PREP OFFICIAL SOLUTION:



As with any Strengthen/Weaken/Assumption problem, here you should identify the conclusion and the premise that supports it. The pundit is concluding that the passage of the tax bill will cost the Republicans the election, and his evidence is that that since the bill was passed Republicans have been trailing in the polls.

An important gap to notice here is one of correlation vs. causation: Republicans are trailing in the polls and a tax bill has been passed, but do you know that the tax bill is the reason for those polls? What if they were already trailing in the polls?

Choice (B) directly attacks this gap. With Assumption questions it can be very helpful to negate answer choices (the Assumption Negation Technique) to see if the negated choice directly weakens the argument (if it does, it's correct). (B), negated, would say that the poll numbers were essentially the same before and after the bill was passed, meaning that the bill itself hasn't had an impact on Republicans' prospects. This directly weakens the pundit's claim that passing the tax bill is going to cost Republicans their majority, so (B) must be correct.

Assumption Negation is a good way to eliminate (A): if you negate (A) it would say that some of the Republicans' competitors supported the bill. Does this cripple the argument? Even if a few challengers supported the bill, that doesn't change the fact that Republicans are still losing in the poll and that many competitors could still strongly oppose the bill. Note: you rarely, if ever, need "none" (or "only" or "all") in an Assumption answer choice - correct assumption answers are something that needs to be true in order for the conclusion to hold, and that level of absoluteness is seldom necessary.

With (C), again note the negated version, which would say that the tax bill is less popular with likely voters than with unlikely voters. If anything this would help the argument by saying that among those who will actually participate in the election, the bill is at its least popular.

(D) may seem close - if the negated version were true and more likely voters supported the bill than opposed it that would seem to say that it's a popular bill, right? - but note that the election isn't about the bill itself, but about which officials will go to congress. If the bill was enough to take some Republican supporters and switch them to the competitors (who have earned votes from other voters for other reasons) that could still mean that the bill was enough to swing the election.

And (E) falls victim to the same problem with (A) - you don't need this bill to be the "most unpopular" bill. As long as it has created a shift in polling from "winning" to "losing" the conclusion still holds.


B says 'substantially different'. 'Different' does not mean'higher'. It could mean that before the bill was passed, the republicans were trailing by 90 points (that also is substantially different). In that case the bill did not overthrow the Republicans( implies that it impacted positively and weakens the conclusion). Similarly, if before the bill was passed , the Republicans were leading by 90 points ( substantially different ), the bill made the difference(strengthening the conclusion)
Hence, B can not be warranted as correct answer choice.

AaronPond,
Conclusion is " the Republicans have likely thrown away their congressional majority in the upcoming election." ===> bill impacted NEGATIVELY. Not just in a causal manner as mentioned in your explanation. B can also show that the bill impacted positively as described in my above reasoning. Please clarify on the mentioned grounds.
Thank you.
Intern
Intern
Joined: 16 Apr 2012
Status:Applying
Affiliations: test
Posts: 26
Own Kudos [?]: 17 [0]
Given Kudos: 120
Location: India
Yawer: Yawer
Concentration: Marketing, Technology
GMAT 1: 700 Q48 V38
WE:Consulting (Internet and New Media)
Send PM
Pundit: By passing the recent tax bill, the Republicans have likely [#permalink]
I disgaree with the OA here although many have explained why B is correct. It says Republicans "consistently trailed their closest competitors by an average of 8 to 10 percentage points.".

Neither the answer nor the question stem explain that Republican's popularity has reduced because of the tax bill. It only says poll numbers are "substantially different". It could both be different positively or negatively.

What if Republicans after gaining majority and before passing the bill were trailing by 20-30%?
Manager
Manager
Joined: 16 Jan 2020
Posts: 77
Own Kudos [?]: 21 [0]
Given Kudos: 38
Location: United States
Schools: Duke '22 (M$)
GMAT 1: 720 Q49 V38
GPA: 3.9
WE:Investment Banking (Other)
Send PM
Re: Pundit: By passing the recent tax bill, the Republicans have likely [#permalink]
GMATGuruNY wrote:
Mo2men wrote:
GMATGuruNY

Why is "C" wrong? how to negate it ?

Thanks


C, negated:
The tax bill is less popular with likely voters than with unlikely voters.
In suggesting that the tax bill is unpopular with likely voters, this negation STRENGTHENS the conclusion that Republicans who passed the bill have likely thrown away their congressional majority in the upcoming election.
Since the correct negation must invalidate the conclusion, eliminate C.



Does answer choice B not create ambiguity? “Were substantially different”. What if they were polling at 20% behind prior to passing the legislation? All the stem states is that the candidates have been trailing 9-10% since the passing. Not 9-10% less than they were prior to the passing of the legislation. They could have been 20% behind other competitors, maybe even 30 prior to the passing of the legislation. The legislation has actually been met warmly.. Its a good question in terms of other tricky answer selections but a GMAT question this is not.
VP
VP
Joined: 10 Jul 2019
Posts: 1392
Own Kudos [?]: 542 [0]
Given Kudos: 1656
Send PM
Pundit: By passing the recent tax bill, the Republicans have likely [#permalink]
This gets to the fundamental difference between the Assumption Q and Strengthen Q.

Stepping back and looking at the structure of the argument is sometimes important in these kinds of questions.

First, you need to get the exact conclusion.

The question writers were very clever with the wording of the Conclusion.

The conclusion is NOT that the Republicans will likely lose their majority in the upcoming election.

The conclusions is that “BY” endorsing this tax bill, the Republicans will likely lose their majority in the upcoming election.

It’s a subtle difference, but the difference is very important to understanding why B is correct.

The pundit is making the argument, essentially, that the act of passing the tax bill will likely CAUSE the Republicans to lose their majority.

The supporting facts used for this conclusion are polling results. We have an “observed fact” that occurred at the same time as the passage of the tax bill. Or, rather, the Republicans passed the tax bill and since that time, they have been losing according to the polls.

The pundit looks at this observed fact/correlation, and then makes the conclusion that the action of passing the tax bill will be the reason why the Republicans will lose a majority in the upcoming election.

Essentially, the author is almost making an assumption with respect to the “observed effect” or fact. He is assuming that the reason why the polls are so low is because the republicans passed the tax bill. And, since the polling numbers are there to back up his argument, the pundit concludes that passing the tax bill is the reason why the Republicans will likely lose their majority.

In order to use the polling numbers as support, backing up his argument, it MUST be the case that the Republicans’ passage of the tax bill lead to these poor poll numbers.

If the tax bill had no effect on the polling numbers, then the author would have no facts upon which he can build his argument. The polling numbers are meaningless and the argument just falls apart. For the author to even use the polling numbers as support for his conclusion, the structure of this argument requires that the tax bill be a reason for the poor numbers.

So, while it would be helpful and strengthen his argument to show that the polling numbers were better for the Republicans before they passed the tax bill, this fact is not absolutely necessary for the conclusion to be made based on the evidence.

What is required for the pundit to use the polling numbers as evidence that passing the tax bill will likely cause the Republicans to lose their majority?

That there was some kind of difference in the polling numbers before and after the passage of the tax bill. Without this difference, the author could not say that the tax bills is the reason for the polling numbers.

If the polling numbers were the same before and after the passage of the tax bill, then how could the author use the polling evidence as support for the conclusion that the tax bill lead to this likelihood of losing the majority?

Tricky question but very well written.



BradyMVP wrote:
GMATGuruNY wrote:
Mo2men wrote:
GMATGuruNY

Why is "C" wrong? how to negate it ?

Thanks


C, negated:
The tax bill is less popular with likely voters than with unlikely voters.
In suggesting that the tax bill is unpopular with likely voters, this negation STRENGTHENS the conclusion that Republicans who passed the bill have likely thrown away their congressional majority in the upcoming election.
Since the correct negation must invalidate the conclusion, eliminate C.



Does answer choice B not create ambiguity? “Were substantially different”. What if they were polling at 20% behind prior to passing the legislation? All the stem states is that the candidates have been trailing 9-10% since the passing. Not 9-10% less than they were prior to the passing of the legislation. They could have been 20% behind other competitors, maybe even 30 prior to the passing of the legislation. The legislation has actually been met warmly.. Its a good question in terms of other tricky answer selections but a GMAT question this is not.


Posted from my mobile device
User avatar
Non-Human User
Joined: 01 Oct 2013
Posts: 17213
Own Kudos [?]: 848 [0]
Given Kudos: 0
Send PM
Re: Pundit: By passing the recent tax bill, the Republicans have likely [#permalink]
Hello from the GMAT Club VerbalBot!

Thanks to another GMAT Club member, I have just discovered this valuable topic, yet it had no discussion for over a year. I am now bumping it up - doing my job. I think you may find it valuable (esp those replies with Kudos).

Want to see all other topics I dig out? Follow me (click follow button on profile). You will receive a summary of all topics I bump in your profile area as well as via email.
GMAT Club Bot
Re: Pundit: By passing the recent tax bill, the Republicans have likely [#permalink]
Moderators:
GMAT Club Verbal Expert
6920 posts
GMAT Club Verbal Expert
238 posts
CR Forum Moderator
832 posts

Powered by phpBB © phpBB Group | Emoji artwork provided by EmojiOne